Substitution leads me to wrong result for the integral $int_0^xlambda e^-lambda xdx$

The name of the pictureThe name of the pictureThe name of the pictureClash Royale CLAN TAG#URR8PPP











up vote
3
down vote

favorite












I have a question that should have been trivial to solve, but for some reason I keep messing up. I need to calculate the following integral:



$$int_0^xlambda e^-lambda xdx$$



I initially tried to solve this with the substitution:



$$u=-lambda x$$



This leads to the following:



$$
beginaligned
x&=-fraculambda\
dx&=-fracdulambda
endaligned$$



My first (wrong) attempt:



$$
beginaligned
int_0^xlambda e^-lambda xdx &= lambdaint_0^frac-ulambda e^ufrac-dulambda\
& = -int_0^frac-ulambda e^udu = -[e^u]_0^frac-ulambda\
&= -[e^frac-ulambda-e^0] = 1 - e^frac-ulambda = 1 - e^x
endaligned
$$



Online tools have shown that the answer is supposed to be $1 - e^-lambda x$ however.



My second slightly modified attempt:



I reuse the steps from my first attempt all the way until I get:



$$-[e^u]_0^frac-ulambda$$



and now I replace the substitution first before continuing:



$$
beginaligned
-[e^u]_0^frac-ulambda &= -[e^-lambda x]_0^x\
& = -[e^-lambda x - e^0]\
& = 1 - e^-lambda x
endaligned
$$



What sorcery is this? I can't seem to recall a rule about substituting the variables back to the original before filling in the limits. Otherwise, what is the point of changing the limits along with the substitution? I must be missing something obvious here, but not obvious enough for me to notice.



Without substitution works without a problem:



$$
beginaligned
int_0^xlambda e^-lambda xdx &= -[e^-lambda x]_0^xdx\
& = -[e^-lambda x - 1]\
&= 1 - e^-lambda x
endaligned
$$



Why was my first attempt wrong?










share|cite|improve this question



















  • 1




    In cases like this, where you only use the substituion for the part of evaluating the indefinite integral, I would suggest to not change the borders of integration. Just leave them as they are and plug in the values into the resubstituted anti-derivative in the end. To change the limits is only a crucial point when you are working with the integral itself without computing the value.
    – mrtaurho
    Aug 11 at 11:11











  • It is not defined. You can not have $dx$ inside if you have $x$ in one of the integral limits.
    – mathreadler
    Aug 11 at 11:13










  • @mrtaurho. But $int_a^b f(x) , dx = x = ky = int_a^b f(ky) , k , dy$ is certainly wrong.
    – md2perpe
    Aug 11 at 11:14






  • 1




    +1 with @mathreadler : $(e^-lambda x)'=-lambda e^-lambda x$, so $int_0^X lambda e^-lambda xrm dx = left[-e^-lambda xright]_0^X = 1-e^-lambda X$.
    – Nicolas FRANCOIS
    Aug 11 at 11:15






  • 1




    It is not a good idea to use the symbol for the integration parameter in the integration boundaries.
    – random
    Aug 11 at 11:16














up vote
3
down vote

favorite












I have a question that should have been trivial to solve, but for some reason I keep messing up. I need to calculate the following integral:



$$int_0^xlambda e^-lambda xdx$$



I initially tried to solve this with the substitution:



$$u=-lambda x$$



This leads to the following:



$$
beginaligned
x&=-fraculambda\
dx&=-fracdulambda
endaligned$$



My first (wrong) attempt:



$$
beginaligned
int_0^xlambda e^-lambda xdx &= lambdaint_0^frac-ulambda e^ufrac-dulambda\
& = -int_0^frac-ulambda e^udu = -[e^u]_0^frac-ulambda\
&= -[e^frac-ulambda-e^0] = 1 - e^frac-ulambda = 1 - e^x
endaligned
$$



Online tools have shown that the answer is supposed to be $1 - e^-lambda x$ however.



My second slightly modified attempt:



I reuse the steps from my first attempt all the way until I get:



$$-[e^u]_0^frac-ulambda$$



and now I replace the substitution first before continuing:



$$
beginaligned
-[e^u]_0^frac-ulambda &= -[e^-lambda x]_0^x\
& = -[e^-lambda x - e^0]\
& = 1 - e^-lambda x
endaligned
$$



What sorcery is this? I can't seem to recall a rule about substituting the variables back to the original before filling in the limits. Otherwise, what is the point of changing the limits along with the substitution? I must be missing something obvious here, but not obvious enough for me to notice.



Without substitution works without a problem:



$$
beginaligned
int_0^xlambda e^-lambda xdx &= -[e^-lambda x]_0^xdx\
& = -[e^-lambda x - 1]\
&= 1 - e^-lambda x
endaligned
$$



Why was my first attempt wrong?










share|cite|improve this question



















  • 1




    In cases like this, where you only use the substituion for the part of evaluating the indefinite integral, I would suggest to not change the borders of integration. Just leave them as they are and plug in the values into the resubstituted anti-derivative in the end. To change the limits is only a crucial point when you are working with the integral itself without computing the value.
    – mrtaurho
    Aug 11 at 11:11











  • It is not defined. You can not have $dx$ inside if you have $x$ in one of the integral limits.
    – mathreadler
    Aug 11 at 11:13










  • @mrtaurho. But $int_a^b f(x) , dx = x = ky = int_a^b f(ky) , k , dy$ is certainly wrong.
    – md2perpe
    Aug 11 at 11:14






  • 1




    +1 with @mathreadler : $(e^-lambda x)'=-lambda e^-lambda x$, so $int_0^X lambda e^-lambda xrm dx = left[-e^-lambda xright]_0^X = 1-e^-lambda X$.
    – Nicolas FRANCOIS
    Aug 11 at 11:15






  • 1




    It is not a good idea to use the symbol for the integration parameter in the integration boundaries.
    – random
    Aug 11 at 11:16












up vote
3
down vote

favorite









up vote
3
down vote

favorite











I have a question that should have been trivial to solve, but for some reason I keep messing up. I need to calculate the following integral:



$$int_0^xlambda e^-lambda xdx$$



I initially tried to solve this with the substitution:



$$u=-lambda x$$



This leads to the following:



$$
beginaligned
x&=-fraculambda\
dx&=-fracdulambda
endaligned$$



My first (wrong) attempt:



$$
beginaligned
int_0^xlambda e^-lambda xdx &= lambdaint_0^frac-ulambda e^ufrac-dulambda\
& = -int_0^frac-ulambda e^udu = -[e^u]_0^frac-ulambda\
&= -[e^frac-ulambda-e^0] = 1 - e^frac-ulambda = 1 - e^x
endaligned
$$



Online tools have shown that the answer is supposed to be $1 - e^-lambda x$ however.



My second slightly modified attempt:



I reuse the steps from my first attempt all the way until I get:



$$-[e^u]_0^frac-ulambda$$



and now I replace the substitution first before continuing:



$$
beginaligned
-[e^u]_0^frac-ulambda &= -[e^-lambda x]_0^x\
& = -[e^-lambda x - e^0]\
& = 1 - e^-lambda x
endaligned
$$



What sorcery is this? I can't seem to recall a rule about substituting the variables back to the original before filling in the limits. Otherwise, what is the point of changing the limits along with the substitution? I must be missing something obvious here, but not obvious enough for me to notice.



Without substitution works without a problem:



$$
beginaligned
int_0^xlambda e^-lambda xdx &= -[e^-lambda x]_0^xdx\
& = -[e^-lambda x - 1]\
&= 1 - e^-lambda x
endaligned
$$



Why was my first attempt wrong?










share|cite|improve this question















I have a question that should have been trivial to solve, but for some reason I keep messing up. I need to calculate the following integral:



$$int_0^xlambda e^-lambda xdx$$



I initially tried to solve this with the substitution:



$$u=-lambda x$$



This leads to the following:



$$
beginaligned
x&=-fraculambda\
dx&=-fracdulambda
endaligned$$



My first (wrong) attempt:



$$
beginaligned
int_0^xlambda e^-lambda xdx &= lambdaint_0^frac-ulambda e^ufrac-dulambda\
& = -int_0^frac-ulambda e^udu = -[e^u]_0^frac-ulambda\
&= -[e^frac-ulambda-e^0] = 1 - e^frac-ulambda = 1 - e^x
endaligned
$$



Online tools have shown that the answer is supposed to be $1 - e^-lambda x$ however.



My second slightly modified attempt:



I reuse the steps from my first attempt all the way until I get:



$$-[e^u]_0^frac-ulambda$$



and now I replace the substitution first before continuing:



$$
beginaligned
-[e^u]_0^frac-ulambda &= -[e^-lambda x]_0^x\
& = -[e^-lambda x - e^0]\
& = 1 - e^-lambda x
endaligned
$$



What sorcery is this? I can't seem to recall a rule about substituting the variables back to the original before filling in the limits. Otherwise, what is the point of changing the limits along with the substitution? I must be missing something obvious here, but not obvious enough for me to notice.



Without substitution works without a problem:



$$
beginaligned
int_0^xlambda e^-lambda xdx &= -[e^-lambda x]_0^xdx\
& = -[e^-lambda x - 1]\
&= 1 - e^-lambda x
endaligned
$$



Why was my first attempt wrong?







integration definite-integrals substitution






share|cite|improve this question















share|cite|improve this question













share|cite|improve this question




share|cite|improve this question








edited Aug 11 at 16:50









AccidentalFourierTransform

1,325627




1,325627










asked Aug 11 at 11:05









Babyburger

1206




1206







  • 1




    In cases like this, where you only use the substituion for the part of evaluating the indefinite integral, I would suggest to not change the borders of integration. Just leave them as they are and plug in the values into the resubstituted anti-derivative in the end. To change the limits is only a crucial point when you are working with the integral itself without computing the value.
    – mrtaurho
    Aug 11 at 11:11











  • It is not defined. You can not have $dx$ inside if you have $x$ in one of the integral limits.
    – mathreadler
    Aug 11 at 11:13










  • @mrtaurho. But $int_a^b f(x) , dx = x = ky = int_a^b f(ky) , k , dy$ is certainly wrong.
    – md2perpe
    Aug 11 at 11:14






  • 1




    +1 with @mathreadler : $(e^-lambda x)'=-lambda e^-lambda x$, so $int_0^X lambda e^-lambda xrm dx = left[-e^-lambda xright]_0^X = 1-e^-lambda X$.
    – Nicolas FRANCOIS
    Aug 11 at 11:15






  • 1




    It is not a good idea to use the symbol for the integration parameter in the integration boundaries.
    – random
    Aug 11 at 11:16












  • 1




    In cases like this, where you only use the substituion for the part of evaluating the indefinite integral, I would suggest to not change the borders of integration. Just leave them as they are and plug in the values into the resubstituted anti-derivative in the end. To change the limits is only a crucial point when you are working with the integral itself without computing the value.
    – mrtaurho
    Aug 11 at 11:11











  • It is not defined. You can not have $dx$ inside if you have $x$ in one of the integral limits.
    – mathreadler
    Aug 11 at 11:13










  • @mrtaurho. But $int_a^b f(x) , dx = x = ky = int_a^b f(ky) , k , dy$ is certainly wrong.
    – md2perpe
    Aug 11 at 11:14






  • 1




    +1 with @mathreadler : $(e^-lambda x)'=-lambda e^-lambda x$, so $int_0^X lambda e^-lambda xrm dx = left[-e^-lambda xright]_0^X = 1-e^-lambda X$.
    – Nicolas FRANCOIS
    Aug 11 at 11:15






  • 1




    It is not a good idea to use the symbol for the integration parameter in the integration boundaries.
    – random
    Aug 11 at 11:16







1




1




In cases like this, where you only use the substituion for the part of evaluating the indefinite integral, I would suggest to not change the borders of integration. Just leave them as they are and plug in the values into the resubstituted anti-derivative in the end. To change the limits is only a crucial point when you are working with the integral itself without computing the value.
– mrtaurho
Aug 11 at 11:11





In cases like this, where you only use the substituion for the part of evaluating the indefinite integral, I would suggest to not change the borders of integration. Just leave them as they are and plug in the values into the resubstituted anti-derivative in the end. To change the limits is only a crucial point when you are working with the integral itself without computing the value.
– mrtaurho
Aug 11 at 11:11













It is not defined. You can not have $dx$ inside if you have $x$ in one of the integral limits.
– mathreadler
Aug 11 at 11:13




It is not defined. You can not have $dx$ inside if you have $x$ in one of the integral limits.
– mathreadler
Aug 11 at 11:13












@mrtaurho. But $int_a^b f(x) , dx = x = ky = int_a^b f(ky) , k , dy$ is certainly wrong.
– md2perpe
Aug 11 at 11:14




@mrtaurho. But $int_a^b f(x) , dx = x = ky = int_a^b f(ky) , k , dy$ is certainly wrong.
– md2perpe
Aug 11 at 11:14




1




1




+1 with @mathreadler : $(e^-lambda x)'=-lambda e^-lambda x$, so $int_0^X lambda e^-lambda xrm dx = left[-e^-lambda xright]_0^X = 1-e^-lambda X$.
– Nicolas FRANCOIS
Aug 11 at 11:15




+1 with @mathreadler : $(e^-lambda x)'=-lambda e^-lambda x$, so $int_0^X lambda e^-lambda xrm dx = left[-e^-lambda xright]_0^X = 1-e^-lambda X$.
– Nicolas FRANCOIS
Aug 11 at 11:15




1




1




It is not a good idea to use the symbol for the integration parameter in the integration boundaries.
– random
Aug 11 at 11:16




It is not a good idea to use the symbol for the integration parameter in the integration boundaries.
– random
Aug 11 at 11:16










2 Answers
2






active

oldest

votes

















up vote
4
down vote



accepted










In your wrong attempt you wrote:
$$int_0^xlambda e^-lambda x mathrm dx = lambdaint_0^colorredfrac-ulambda e^ufrac-mathrm dulambda$$
when in fact you should have written:
$$int_0^xlambda e^-lambda x mathrm dx = lambdaint_0^colorredu e^ufrac-mathrm dulambda$$



When changing the bounds, ask yourself "when $x$ = lower bound, what is $u$? when $x$ = upper bound, what is $u$?"



The answer is "when $x=0$, $u=0$; when $x=x$, $u=u$".






share|cite|improve this answer




















  • But it is still wrong (and confusing, see the OP) to write $int_0^x f(x)rm dx$...
    – Nicolas FRANCOIS
    Aug 11 at 11:17










  • Well $displaystyle int_0^x f(x) mathrm dx$ isn't necessarily wrong.
    – Kenny Lau
    Aug 11 at 11:27










  • For the sanity of students, it SHOULD be :-)
    – Nicolas FRANCOIS
    Aug 11 at 11:29










  • @KennyLau yes it is wrong. It makes no sense whatsoever.
    – mathreadler
    Aug 11 at 14:52











  • Ok, for some reason that took a while to sink in and now I feel stupid because how obvious it is (as I expected it would be). I shouldn't have written the upper bound in terms of x=..., but keep it in the form of u=.... Thanks a lot!
    – Babyburger
    Aug 11 at 16:49

















up vote
2
down vote













One mistake at the start with the substitution $u=-lambda x$ you should get $$dx = -fracdulambda $$
but for some reason you get
$$dx = -fracdux$$



If you keep going from that spot (and change integral limit to for example $X$ as proposed by Nicolas Francois) you should be able to solve it.






share|cite|improve this answer
















  • 1




    Thanks for pointing that out! It was supposed to be lambda, but I made a typing mistake with my OP. I edited it.
    – Babyburger
    Aug 11 at 16:29










Your Answer




StackExchange.ifUsing("editor", function ()
return StackExchange.using("mathjaxEditing", function ()
StackExchange.MarkdownEditor.creationCallbacks.add(function (editor, postfix)
StackExchange.mathjaxEditing.prepareWmdForMathJax(editor, postfix, [["$", "$"], ["\\(","\\)"]]);
);
);
, "mathjax-editing");

StackExchange.ready(function()
var channelOptions =
tags: "".split(" "),
id: "69"
;
initTagRenderer("".split(" "), "".split(" "), channelOptions);

StackExchange.using("externalEditor", function()
// Have to fire editor after snippets, if snippets enabled
if (StackExchange.settings.snippets.snippetsEnabled)
StackExchange.using("snippets", function()
createEditor();
);

else
createEditor();

);

function createEditor()
StackExchange.prepareEditor(
heartbeatType: 'answer',
convertImagesToLinks: true,
noModals: false,
showLowRepImageUploadWarning: true,
reputationToPostImages: 10,
bindNavPrevention: true,
postfix: "",
noCode: true, onDemand: true,
discardSelector: ".discard-answer"
,immediatelyShowMarkdownHelp:true
);



);













 

draft saved


draft discarded


















StackExchange.ready(
function ()
StackExchange.openid.initPostLogin('.new-post-login', 'https%3a%2f%2fmath.stackexchange.com%2fquestions%2f2879289%2fsubstitution-leads-me-to-wrong-result-for-the-integral-int-0x-lambda-e-lam%23new-answer', 'question_page');

);

Post as a guest






























2 Answers
2






active

oldest

votes








2 Answers
2






active

oldest

votes









active

oldest

votes






active

oldest

votes








up vote
4
down vote



accepted










In your wrong attempt you wrote:
$$int_0^xlambda e^-lambda x mathrm dx = lambdaint_0^colorredfrac-ulambda e^ufrac-mathrm dulambda$$
when in fact you should have written:
$$int_0^xlambda e^-lambda x mathrm dx = lambdaint_0^colorredu e^ufrac-mathrm dulambda$$



When changing the bounds, ask yourself "when $x$ = lower bound, what is $u$? when $x$ = upper bound, what is $u$?"



The answer is "when $x=0$, $u=0$; when $x=x$, $u=u$".






share|cite|improve this answer




















  • But it is still wrong (and confusing, see the OP) to write $int_0^x f(x)rm dx$...
    – Nicolas FRANCOIS
    Aug 11 at 11:17










  • Well $displaystyle int_0^x f(x) mathrm dx$ isn't necessarily wrong.
    – Kenny Lau
    Aug 11 at 11:27










  • For the sanity of students, it SHOULD be :-)
    – Nicolas FRANCOIS
    Aug 11 at 11:29










  • @KennyLau yes it is wrong. It makes no sense whatsoever.
    – mathreadler
    Aug 11 at 14:52











  • Ok, for some reason that took a while to sink in and now I feel stupid because how obvious it is (as I expected it would be). I shouldn't have written the upper bound in terms of x=..., but keep it in the form of u=.... Thanks a lot!
    – Babyburger
    Aug 11 at 16:49














up vote
4
down vote



accepted










In your wrong attempt you wrote:
$$int_0^xlambda e^-lambda x mathrm dx = lambdaint_0^colorredfrac-ulambda e^ufrac-mathrm dulambda$$
when in fact you should have written:
$$int_0^xlambda e^-lambda x mathrm dx = lambdaint_0^colorredu e^ufrac-mathrm dulambda$$



When changing the bounds, ask yourself "when $x$ = lower bound, what is $u$? when $x$ = upper bound, what is $u$?"



The answer is "when $x=0$, $u=0$; when $x=x$, $u=u$".






share|cite|improve this answer




















  • But it is still wrong (and confusing, see the OP) to write $int_0^x f(x)rm dx$...
    – Nicolas FRANCOIS
    Aug 11 at 11:17










  • Well $displaystyle int_0^x f(x) mathrm dx$ isn't necessarily wrong.
    – Kenny Lau
    Aug 11 at 11:27










  • For the sanity of students, it SHOULD be :-)
    – Nicolas FRANCOIS
    Aug 11 at 11:29










  • @KennyLau yes it is wrong. It makes no sense whatsoever.
    – mathreadler
    Aug 11 at 14:52











  • Ok, for some reason that took a while to sink in and now I feel stupid because how obvious it is (as I expected it would be). I shouldn't have written the upper bound in terms of x=..., but keep it in the form of u=.... Thanks a lot!
    – Babyburger
    Aug 11 at 16:49












up vote
4
down vote



accepted







up vote
4
down vote



accepted






In your wrong attempt you wrote:
$$int_0^xlambda e^-lambda x mathrm dx = lambdaint_0^colorredfrac-ulambda e^ufrac-mathrm dulambda$$
when in fact you should have written:
$$int_0^xlambda e^-lambda x mathrm dx = lambdaint_0^colorredu e^ufrac-mathrm dulambda$$



When changing the bounds, ask yourself "when $x$ = lower bound, what is $u$? when $x$ = upper bound, what is $u$?"



The answer is "when $x=0$, $u=0$; when $x=x$, $u=u$".






share|cite|improve this answer












In your wrong attempt you wrote:
$$int_0^xlambda e^-lambda x mathrm dx = lambdaint_0^colorredfrac-ulambda e^ufrac-mathrm dulambda$$
when in fact you should have written:
$$int_0^xlambda e^-lambda x mathrm dx = lambdaint_0^colorredu e^ufrac-mathrm dulambda$$



When changing the bounds, ask yourself "when $x$ = lower bound, what is $u$? when $x$ = upper bound, what is $u$?"



The answer is "when $x=0$, $u=0$; when $x=x$, $u=u$".







share|cite|improve this answer












share|cite|improve this answer



share|cite|improve this answer










answered Aug 11 at 11:15









Kenny Lau

19k2157




19k2157











  • But it is still wrong (and confusing, see the OP) to write $int_0^x f(x)rm dx$...
    – Nicolas FRANCOIS
    Aug 11 at 11:17










  • Well $displaystyle int_0^x f(x) mathrm dx$ isn't necessarily wrong.
    – Kenny Lau
    Aug 11 at 11:27










  • For the sanity of students, it SHOULD be :-)
    – Nicolas FRANCOIS
    Aug 11 at 11:29










  • @KennyLau yes it is wrong. It makes no sense whatsoever.
    – mathreadler
    Aug 11 at 14:52











  • Ok, for some reason that took a while to sink in and now I feel stupid because how obvious it is (as I expected it would be). I shouldn't have written the upper bound in terms of x=..., but keep it in the form of u=.... Thanks a lot!
    – Babyburger
    Aug 11 at 16:49
















  • But it is still wrong (and confusing, see the OP) to write $int_0^x f(x)rm dx$...
    – Nicolas FRANCOIS
    Aug 11 at 11:17










  • Well $displaystyle int_0^x f(x) mathrm dx$ isn't necessarily wrong.
    – Kenny Lau
    Aug 11 at 11:27










  • For the sanity of students, it SHOULD be :-)
    – Nicolas FRANCOIS
    Aug 11 at 11:29










  • @KennyLau yes it is wrong. It makes no sense whatsoever.
    – mathreadler
    Aug 11 at 14:52











  • Ok, for some reason that took a while to sink in and now I feel stupid because how obvious it is (as I expected it would be). I shouldn't have written the upper bound in terms of x=..., but keep it in the form of u=.... Thanks a lot!
    – Babyburger
    Aug 11 at 16:49















But it is still wrong (and confusing, see the OP) to write $int_0^x f(x)rm dx$...
– Nicolas FRANCOIS
Aug 11 at 11:17




But it is still wrong (and confusing, see the OP) to write $int_0^x f(x)rm dx$...
– Nicolas FRANCOIS
Aug 11 at 11:17












Well $displaystyle int_0^x f(x) mathrm dx$ isn't necessarily wrong.
– Kenny Lau
Aug 11 at 11:27




Well $displaystyle int_0^x f(x) mathrm dx$ isn't necessarily wrong.
– Kenny Lau
Aug 11 at 11:27












For the sanity of students, it SHOULD be :-)
– Nicolas FRANCOIS
Aug 11 at 11:29




For the sanity of students, it SHOULD be :-)
– Nicolas FRANCOIS
Aug 11 at 11:29












@KennyLau yes it is wrong. It makes no sense whatsoever.
– mathreadler
Aug 11 at 14:52





@KennyLau yes it is wrong. It makes no sense whatsoever.
– mathreadler
Aug 11 at 14:52













Ok, for some reason that took a while to sink in and now I feel stupid because how obvious it is (as I expected it would be). I shouldn't have written the upper bound in terms of x=..., but keep it in the form of u=.... Thanks a lot!
– Babyburger
Aug 11 at 16:49




Ok, for some reason that took a while to sink in and now I feel stupid because how obvious it is (as I expected it would be). I shouldn't have written the upper bound in terms of x=..., but keep it in the form of u=.... Thanks a lot!
– Babyburger
Aug 11 at 16:49










up vote
2
down vote













One mistake at the start with the substitution $u=-lambda x$ you should get $$dx = -fracdulambda $$
but for some reason you get
$$dx = -fracdux$$



If you keep going from that spot (and change integral limit to for example $X$ as proposed by Nicolas Francois) you should be able to solve it.






share|cite|improve this answer
















  • 1




    Thanks for pointing that out! It was supposed to be lambda, but I made a typing mistake with my OP. I edited it.
    – Babyburger
    Aug 11 at 16:29














up vote
2
down vote













One mistake at the start with the substitution $u=-lambda x$ you should get $$dx = -fracdulambda $$
but for some reason you get
$$dx = -fracdux$$



If you keep going from that spot (and change integral limit to for example $X$ as proposed by Nicolas Francois) you should be able to solve it.






share|cite|improve this answer
















  • 1




    Thanks for pointing that out! It was supposed to be lambda, but I made a typing mistake with my OP. I edited it.
    – Babyburger
    Aug 11 at 16:29












up vote
2
down vote










up vote
2
down vote









One mistake at the start with the substitution $u=-lambda x$ you should get $$dx = -fracdulambda $$
but for some reason you get
$$dx = -fracdux$$



If you keep going from that spot (and change integral limit to for example $X$ as proposed by Nicolas Francois) you should be able to solve it.






share|cite|improve this answer












One mistake at the start with the substitution $u=-lambda x$ you should get $$dx = -fracdulambda $$
but for some reason you get
$$dx = -fracdux$$



If you keep going from that spot (and change integral limit to for example $X$ as proposed by Nicolas Francois) you should be able to solve it.







share|cite|improve this answer












share|cite|improve this answer



share|cite|improve this answer










answered Aug 11 at 11:19









mathreadler

13.9k72058




13.9k72058







  • 1




    Thanks for pointing that out! It was supposed to be lambda, but I made a typing mistake with my OP. I edited it.
    – Babyburger
    Aug 11 at 16:29












  • 1




    Thanks for pointing that out! It was supposed to be lambda, but I made a typing mistake with my OP. I edited it.
    – Babyburger
    Aug 11 at 16:29







1




1




Thanks for pointing that out! It was supposed to be lambda, but I made a typing mistake with my OP. I edited it.
– Babyburger
Aug 11 at 16:29




Thanks for pointing that out! It was supposed to be lambda, but I made a typing mistake with my OP. I edited it.
– Babyburger
Aug 11 at 16:29

















 

draft saved


draft discarded















































 


draft saved


draft discarded














StackExchange.ready(
function ()
StackExchange.openid.initPostLogin('.new-post-login', 'https%3a%2f%2fmath.stackexchange.com%2fquestions%2f2879289%2fsubstitution-leads-me-to-wrong-result-for-the-integral-int-0x-lambda-e-lam%23new-answer', 'question_page');

);

Post as a guest













































































Popular posts from this blog

How to check contact read email or not when send email to Individual?

Bahrain

Postfix configuration issue with fips on centos 7; mailgun relay